diff --git a/Advanced/Introduction to Quantum/main.tex b/Advanced/Introduction to Quantum/main.tex index 7ed68d9..e186af3 100755 --- a/Advanced/Introduction to Quantum/main.tex +++ b/Advanced/Introduction to Quantum/main.tex @@ -17,8 +17,7 @@ % use the [solutions] flag to show solutions. \documentclass[ solutions, - singlenumbering, - unfinished + singlenumbering ]{../../resources/ormc_handout} \usepackage{../../resources/macros} @@ -46,53 +45,25 @@ \input{parts/03.00 logic gates} \input{parts/03.01 quantum gates} -\end{document} + %\section{Superdense Coding} + %TODO + %\vfill + %\pagebreak -\problem{} -The SWAP gate swaps two bits: $\text{SWAP}\ket{ab} = \ket{ba}$. \par -Find its matrix. - -\begin{solution} - \begin{equation*} - \text{SWAP} = \begin{bmatrix} - 1 & 0 & 0 & 0 \\ - 0 & 0 & 1 & 0 \\ - 0 & 1 & 0 & 0 \\ - 0 & 0 & 0 & 1 \\ - \end{bmatrix} - \end{equation*} -\end{solution} - -\vfill + %\section{Quantum Error Correction} + %TODO: no cloning theorem, bit flip code + %\vfill + %\pagebreak + %\section{Quantum Teleportation} + %TODO + %\vfill + %\pagebreak + + %\section{One Real Qubit} + % No problems, appendix. + % bloch sphere, etc. - - - - - -% \problem{} -% The $T$ gate is a three-bit gate that inverts its right bit iff its left and middle inputs are both $\ket{1}$. \par -% In other words, $T\ket{11x} = \ket{11}\ket{\text{not } x}$, and $T\ket{abx} = \ket{abx}$ for all other inputs. \par -% Find the $T$ gate's matrix. \par -% \note{ -% This gate is particularly interesting because it's a \textit{universal quantum gate}: \\ -% like NOR and NAND in classical logic, any quantum gate may emulated by only applying $T$ gates. -% } -% -% \begin{solution} -% \begin{equation*} -% \text{T} = \begin{bmatrix} -% 1 & 0 & 0 & 0 & 0 & 0 & 0 & 0 \\ -% 0 & 1 & 0 & 0 & 0 & 0 & 0 & 0 \\ -% 0 & 0 & 1 & 0 & 0 & 0 & 0 & 0 \\ -% 0 & 0 & 0 & 1 & 0 & 0 & 0 & 0 \\ -% 0 & 0 & 0 & 0 & 1 & 0 & 0 & 0 \\ -% 0 & 0 & 0 & 0 & 0 & 1 & 0 & 0 \\ -% 0 & 0 & 0 & 0 & 0 & 0 & 0 & 1 \\ -% 0 & 0 & 0 & 0 & 0 & 0 & 1 & 0 \\ -% \end{bmatrix} -% \end{equation*} -% \end{solution} \ No newline at end of file +\end{document} \ No newline at end of file diff --git a/Advanced/Introduction to Quantum/parts/03.00 logic gates.tex b/Advanced/Introduction to Quantum/parts/03.00 logic gates.tex index 7ab5fde..7ec920a 100644 --- a/Advanced/Introduction to Quantum/parts/03.00 logic gates.tex +++ b/Advanced/Introduction to Quantum/parts/03.00 logic gates.tex @@ -290,8 +290,7 @@ We could draw the above transformation as a combination $X$ and $I$ (identity) g \end{tikzpicture} \end{center} -We can even omit the $I$ gate, since we now know that transforms affect the whole state. \par -Of course, empty spaces always imply an $I$ gate. +We can even omit the $I$ gate, since we now know that transforms affect the whole state: \par \begin{center} \begin{tikzpicture}[scale=0.8] \node[qubit] (a) at (0, 0) {$\ket{0}$}; diff --git a/Advanced/Introduction to Quantum/parts/03.01 quantum gates.tex b/Advanced/Introduction to Quantum/parts/03.01 quantum gates.tex index a9572cc..056804a 100644 --- a/Advanced/Introduction to Quantum/parts/03.01 quantum gates.tex +++ b/Advanced/Introduction to Quantum/parts/03.01 quantum gates.tex @@ -105,9 +105,8 @@ Find the matrix that applies the cnot gate. \vfill \generic{Remark:} -Note that a quantum gate is fully defined by the place it maps -our basis states $\ket{0}$ and $\ket{1}$ (or,$\ket{00...0}$ through $\ket{11...1}$ for multi-qubit gates). -This directly follows from \ref{qgateislinear}. +As we just saw, a quantum gate is fully defined by the place it maps our basis states $\ket{0}$ and $\ket{1}$ \par +(or, $\ket{00...0}$ through $\ket{11...1}$ for multi-qubit gates). This directly follows from \ref{qgateislinear}. \pagebreak @@ -172,28 +171,18 @@ If we measure the result of \ref{applycnot}, what are the probabilities of getti %\vfill + \definition{} -The \textit{Hadamard Gate} $H$, is given by the following matrix: \par +The \textit{Hadamard Gate} is given by the following matrix: \par \begin{equation*} H = \frac{1}{\sqrt{2}}\begin{bmatrix} 1 & 1 \\ 1 & -1 \end{bmatrix} \end{equation*} -\note[Note]{Note that we divide by $\sqrt{2}$, since $H$ must be orthonormal} +\note{Note that we divide by $\sqrt{2}$, since $H$ must be orthonormal.} - -\problem{} -What is $HH$? \par -Using this result, find $H^{-1}$. - -\begin{solution} - $HH = I$, so $H^{-1} = H$ -\end{solution} - -\vfill - -\begin{ORMCbox}{Review: Matrix Multiplication}{black!10!white}{black} +\begin{ORMCbox}{Review: Matrix Multiplication}{black!10!white}{black!65!white} Matrix multiplication works as follows: \begin{equation*} @@ -236,4 +225,25 @@ Using this result, find $H^{-1}$. This is exactly the first column of the matrix product. \end{ORMCbox} + +\problem{} +What is $HH$? \par +Using this result, find $H^{-1}$. + +\begin{solution} + $HH = I$, so $H^{-1} = H$ +\end{solution} + +\vfill + +\problem{} +What are $H\ket{0}$ and $H\ket{1}$? \par +Are these states superpositions? + +\begin{solution} + $H\ket{0} = \frac{1}{\sqrt{2}}\bigl(\ket{0} + \ket{1}\bigr)$ and $H\ket{1} = \frac{1}{\sqrt{2}}\bigl(\ket{0} - \ket{1}\bigr)$ \par + Both of these are superpositions. +\end{solution} + +\vfill \pagebreak